Did I apply this limit comparison test correctly?

Click For Summary
SUMMARY

The forum discussion centers on applying the limit comparison test to determine the convergence or divergence of the series Σ((5n^3)+1)/((2^n)((n^3)+n+1)). The limit comparison test states that if the limit of An/Bn equals a constant c (where 0 < c < ∞), then both series either converge or diverge together. The user expressed uncertainty regarding the choice of the comparison series Bn and the behavior of An as n approaches infinity. Feedback from other users emphasized the importance of clearly typed solutions over screenshots for effective assistance.

PREREQUISITES
  • Understanding of the limit comparison test in series convergence
  • Familiarity with infinite series notation and summation
  • Knowledge of asymptotic behavior of functions as n approaches infinity
  • Proficiency in LaTeX for formatting mathematical expressions
NEXT STEPS
  • Review the limit comparison test with examples from calculus textbooks
  • Practice identifying appropriate comparison series for complex functions
  • Learn how to format mathematical expressions using LaTeX
  • Explore convergence tests for series, such as the ratio test and root test
USEFUL FOR

Students studying calculus, particularly those focusing on series convergence, as well as educators and tutors assisting with mathematical problem-solving techniques.

Randall
Messages
22
Reaction score
0

Homework Statement


Use the limit comparison test to show the series converges or diverges: Sum from n=1 to infinity of ((5n^3)+1)/((2^n)((n^3)+n+1))

Homework Equations


suppose Sum An and Sum Bn are two positive series. Let lim as n goes to infinity of An/Bn = c: 1) if 0<c<inifinity then either both converge or both diverge; 2) if c=0 and Sum Bn converges, so does Sum An; 3) if c=infinity and Sum Bn diverges, so does Sum An

The Attempt at a Solution


See attached - series An is very complex. I'm not sure it goes to infinity like I assumed, nor am I sure that I picked an appropriate Bn equation. Please help thanks.
 

Attachments

  • limit_comparison_test_2.jpg
    limit_comparison_test_2.jpg
    31.6 KB · Views: 487
Physics news on Phys.org
Randall said:

Homework Statement


Use the limit comparison test to show the series converges or diverges: Sum from n=1 to infinity of ((5n^3)+1)/((2^n)((n^3)+n+1))

Homework Equations


suppose Sum An and Sum Bn are two positive series. Let lim as n goes to infinity of An/Bn = c: 1) if 0<c<inifinity then either both converge or both diverge; 2) if c=0 and Sum Bn converges, so does Sum An; 3) if c=infinity and Sum Bn diverges, so does Sum An

The Attempt at a Solution


See attached - series An is very complex. I'm not sure it goes to infinity like I assumed, nor am I sure that I picked an appropriate Bn equation. Please help thanks.

I wish you would take the trouble to type out the answer, rather than giving it in a rather messy screen shot. (For more on this issue, see the pinned posting "Guidelines for Students and Helpers", by Vela.) However, as far as I can see, what you wrote is not in any way relevant to the problem. However, I won't say more now, since I think you need to go back and think carefully about what you are doing.
 
Last edited by a moderator:
Ray Vickson said:
I wish you would take the trouble to type out the answer, rather than giving it in a rather messy screen shot.
I agree. In addition to what Ray said, posting an image of your work rather than a typed version of your work prevents us from pointing out exactly where your mistake is.

I am closing this thread. For tips on how to use LaTeX to format mathematics expressions, see https://www.physicsforums.com/help/latexhelp/.

Ray Vickson said:
(For more on this issue, see the pinned posting "Guidelines for Students and Helpers", by Vela.) However, as far as I can see, what you wrote is not in any way relevant to the problem. However, I won't say more now, since I think you need to go back and think carefully about what you are doing.
I see several mistakes in your work, one of which is saying that ∞/∞ = ∞.
 
Question: A clock's minute hand has length 4 and its hour hand has length 3. What is the distance between the tips at the moment when it is increasing most rapidly?(Putnam Exam Question) Answer: Making assumption that both the hands moves at constant angular velocities, the answer is ## \sqrt{7} .## But don't you think this assumption is somewhat doubtful and wrong?

Similar threads

  • · Replies 10 ·
Replies
10
Views
3K
  • · Replies 6 ·
Replies
6
Views
1K
  • · Replies 1 ·
Replies
1
Views
2K
Replies
29
Views
2K
  • · Replies 6 ·
Replies
6
Views
2K
  • · Replies 2 ·
Replies
2
Views
2K
  • · Replies 2 ·
Replies
2
Views
2K
  • · Replies 4 ·
Replies
4
Views
1K
  • · Replies 14 ·
Replies
14
Views
2K
  • · Replies 6 ·
Replies
6
Views
2K